LSAT and Law School Admissions Forum

Get expert LSAT preparation and law school admissions advice from PowerScore Test Preparation.

User avatar
 Dave Killoran
PowerScore Staff
  • PowerScore Staff
  • Posts: 5852
  • Joined: Mar 25, 2011
|
#27110
Complete Question Explanation
(The complete setup for this game can be found here: lsat/viewtopic.php?t=49)

The correct answer choice is (E)

Oddly, in this Cannot Be True question the Local condition has no effect on the correct answer. From the discussion of inferences, we know that when P is carried then T must be carried. Thus, the statement in answer choice (E), that P is carried but T is not carried, cannot be true and answer choice (E) is correct. Clearly, the test makers felt this inference was extremely challenging to make, and so it is tested repeatedly in this game.

Get the most out of your LSAT Prep Plus subscription.

Analyze and track your performance with our Testing and Analytics Package.